11

二項係数の3乗が入った級数その2

384
0
$$$$

この記事では, 二項係数の3乗が入った2つの級数

\begin{align*} \sum_{n=0}^{\infty}\frac{\binom{2n}n^3}{2^{6n}}\sum_{k=1}^n\frac 1k&=\frac{2\pi}{\Gamma\left(\frac 34\right)^4}\left(\frac{\pi}3-\ln 2\right)\\ \sum_{n=0}^{\infty}\frac{\binom{2n}n^3}{2^{6n}}\sum_{k=1}^n\frac 1{k-\frac 12}&=\frac{\pi^2}{3\Gamma\left(\frac 34\right)^4} \end{align*}

に証明を与える. 用いる定理は以下である.

Dixonの恒等式

\begin{align*} \sum_{n=0}^{\infty}\frac{(a)_n(b)_n(c)_n}{n!(1+a-b)_n(1+a-c)_n}=\frac{\Gamma\left(1+\frac a2\right)\Gamma(1+a-b)\Gamma(1+a-c)\Gamma\left(1+\frac a2-b-c\right)}{\Gamma(1+a)\Gamma\left(1+\frac a2-b\right)\Gamma\left(1+\frac a2-c\right)\Gamma(1+a-b-c)} \end{align*}

まず, これを$a$に関して偏微分すると,

\begin{align*} &\sum_{n=0}^{\infty}\frac{(a)_n(b)_n(c)_n}{n!(1+a-b)_n(1+a-c)_n}\sum_{k=1}^n\left(\frac 1{k+a-1}-\frac 1{k+a-b}-\frac 1{k+a-c}\right)\\ &=\frac{\Gamma\left(1+\frac a2\right)\Gamma(1+a-b)\Gamma(1+a-c)\Gamma\left(1+\frac a2-b-c\right)}{\Gamma(1+a)\Gamma\left(1+\frac a2-b\right)\Gamma\left(1+\frac a2-c\right)\Gamma(1+a-b-c)}\left(\frac 12\left(\psi\left(1+\frac a2\right)-\psi\left(1+\frac a2-b\right)-\psi\left(1+\frac a2-c\right)+\psi\left(1+\frac a2-b-c\right)\right)-\psi(1+a)+\psi(1+a-b)+\psi(1+a-c)-\psi(1+a-b-c)\right) \end{align*}

ここに$a=b=c=\frac 12$を代入すると,

\begin{align*} \sum_{n=0}^{\infty}\frac{\binom{2n}n^3}{2^{6n}}\sum_{k=1}^n\left(\frac 1{k-\frac 12}-\frac 2k\right)&=\frac{\pi}{\Gamma\left(\frac 34\right)^4}\left(\frac 12\left(\psi\left(\frac 54\right)-2\psi\left(\frac 34\right)+\psi\left(\frac 14\right)\right)-\psi\left(\frac 32\right)+2\psi(1)-\psi\left(\frac 12\right)\right)\\ &=\frac{\pi}{\Gamma\left(\frac 34\right)^4}(4\ln 2-\pi) \end{align*}

また, Dixonの恒等式を$b$に関して偏微分すると,

\begin{align*} &\sum_{n=0}^{\infty}\frac{(a)_n(b)_n(c)_n}{n!(1+a-b)_n(1+a-c)_n}\sum_{k=1}^n\left(\frac 1{k+b-1}+\frac 1{k+a-b}\right)\\ &=\frac{\Gamma\left(1+\frac a2\right)\Gamma(1+a-b)\Gamma(1+a-c)\Gamma\left(1+\frac a2-b-c\right)}{\Gamma(1+a)\Gamma\left(1+\frac a2-b\right)\Gamma\left(1+\frac a2-c\right)\Gamma(1+a-b-c)}\left(\psi\left(1+\frac a2-b\right)-\psi\left(1+\frac a2-b-c\right)-\psi(1+a-b)+\psi(1+a-b-c)\right) \end{align*}

これに$a=b=c=\frac 12$を代入すると,

\begin{align*} \sum_{n=0}^{\infty}\frac{\binom{2n}n^3}{2^{6n}}\sum_{k=1}^n\left(\frac 1{k-\frac 12}+\frac 1k\right)&=\frac{\pi}{\Gamma\left(\frac 34\right)^4}\left(\psi\left(\frac 34\right)-\psi\left(\frac 14\right)-\psi(1)+\psi\left(\frac 12\right)\right)\\ &=\frac{\pi}{\Gamma\left(\frac 34\right)^4}(\pi-2\ln 2) \end{align*}

よって,

\begin{align*} \sum_{n=0}^{\infty}\frac{\binom{2n}n^3}{2^{6n}}\sum_{k=1}^n\left(\frac 1{k-\frac 12}-\frac 2k\right)&=\frac{\pi}{\Gamma\left(\frac 34\right)^4}(4\ln 2-\pi)\\ \sum_{n=0}^{\infty}\frac{\binom{2n}n^3}{2^{6n}}\sum_{k=1}^n\left(\frac 1{k-\frac 12}+\frac 1k\right)&=\frac{\pi}{\Gamma\left(\frac 34\right)^4}(\pi-2\ln 2) \end{align*}

の2つを用いて,

\begin{align*} \sum_{n=0}^{\infty}\frac{\binom{2n}n^3}{2^{6n}}\sum_{k=1}^n\frac 1k&=\frac{2\pi}{\Gamma\left(\frac 34\right)^4}\left(\frac{\pi}3-\ln 2\right)\\ \sum_{n=0}^{\infty}\frac{\binom{2n}n^3}{2^{6n}}\sum_{k=1}^n\frac 1{k-\frac 12}&=\frac{\pi^2}{3\Gamma\left(\frac 34\right)^4} \end{align*}

と求められる.

投稿日:20231026

この記事を高評価した人

高評価したユーザはいません

この記事に送られたバッジ

バッジはありません。

投稿者

Wataru
Wataru
404
25439
超幾何関数, 直交関数, 多重ゼータ値などに興味があります

コメント

他の人のコメント

コメントはありません。
読み込み中...
読み込み中